陈文登考研数学辅导书(附带详细答案本)

上传人:缘*** 文档编号:333287532 上传时间:2022-09-01 格式:PDF 页数:114 大小:15.16MB
返回 下载 相关 举报
陈文登考研数学辅导书(附带详细答案本)_第1页
第1页 / 共114页
陈文登考研数学辅导书(附带详细答案本)_第2页
第2页 / 共114页
陈文登考研数学辅导书(附带详细答案本)_第3页
第3页 / 共114页
陈文登考研数学辅导书(附带详细答案本)_第4页
第4页 / 共114页
陈文登考研数学辅导书(附带详细答案本)_第5页
第5页 / 共114页
点击查看更多>>
资源描述

《陈文登考研数学辅导书(附带详细答案本)》由会员分享,可在线阅读,更多相关《陈文登考研数学辅导书(附带详细答案本)(114页珍藏版)》请在金锄头文库上搜索。

1、函 数 极 限 连 续一.填空题fan=p biAi.设 T*1,则 =+;n )=解.a+/+H+J/+曷+JL +2+f-厂二+2+A 4-f-Ma4-J4-t 1-M 4-2 M3+JV+JK +l Jz+*+l JH1 4-J4-t14-2+A+M 1+2+4 +R 1+2+A+,所以 R+#+R ja 4-Jt+l W+R+2 M2 4-JH+X +l14-24-A+M_ 2 y 1J H1+x+JV3 4-JV 4-M 2,(n)1+2+A 4-JC _ _ 2 IJ a4-w4-l JI1+J4-1 2,(n)解.ff(x)=1.I配(而+3 -加-)f a n 7 K -G)=

2、l m&*+雨_*解.L n+3-气+_2=+4-石5.-=Jl9 JI=-所以 k-l=1 9 9 0,k =1 9 9 1;*181二.单项选择题1.设/X x)和(x)在(一,+)内有定义,F(x)为连续函数,且 f(x)0,(x)有间断点,则(a)(x)必有间断点(b)J)?必有间断点(c)f (x)必有间断点(d)/U)必有间断点X77例反(a)蟀fx)=1,贝 I L/(x)=l,1OrLr则11u=3例反例反XXbA(才(d)反 设 g(x)=/(X)在(一,+)内连续,贝 U (x)=g(x)f(x)在(一,+)内连续,矛 盾.所 以(d)是答案.2.设函数/外二大1 8axy

3、 ,则 f(x)是(a)偶函数(b)无界函数(c)周期函数(d)单调函数解.(b)是答案.3.极限2+l 1ja x(j+I)a J的值是眶+2,x l1(a)0 (b)1 (c)2 (d)不存在+尸1 J解.ta x2a+2*x l1+A诗 W+A小闺l-小 卜,所以(b)为答案.d +D+D$Y4.设一(x+D,则 a的值为(a)1 (b)2 (c)V 5 (d)均不对口+铲3+/小(x+D*/C o x+D5/?解8 =.*卅=巴 一 声 萨7 K 小+1/如(0jr=0 x 0,b 0解.K=lfM,C=bla4.求下列函数的间断点并判别类型“2 _ 1 2 1/(P*)=5=1 /(

4、P-)=鸣=-I解.2*+1 ,2*4-1所以x=0 为第一类间断点.显 然/(l+O)=-L/(I-0)=1,所以x=1 为第一类间断点;/(-n-q)=-L 所以x=-1 为第一类间断点.X(2JT 4-x)/(*)=2 T 3向-X-I *-I x0解.f(+0)=-s in l,f(-0)=0.所以x=0 为第一类跳跃间断点;Ian y(x)=lim sn J.*1-l不存在.所以x=1 为第二类间断点;/不存在,而2co3X 2,所以Xo 为第一类可去间断点;fan 项 *)=8至 2cosx,(,k 1,2,)所 以 x=2 为第二类无穷间断点./但=-1x+na x _(a+力血

5、 x)5.设 x,且 x=0 是 f(x)的可去间断点.求解.x=0是f(x)的可去间断点,要求觇(Vl+ah x+ajpi*Vt+sm x+ma x-(a +jjan x)G 存在.所以-Qz+4*切=0.所以%1+sh x+sh3x-(a-l-j0sai x)30=,*Jl4向*4 x而 x)G(t-+(1-25)热 x+Q-jJ3)nJ m _ 1-a;_*x+热;x+(a+/sin x)l+所以=1._ 也+-(a+A-x)S i*sn XJ。-炉a +4声=*a x.l+a i x+sh吸 +(l+/a x)上式极限存在,必须 2.、.fan k,+7x4+2),-*=A 八6.设-

6、,b 0,求a,b的值.1解.上式极限存在,必须a=5(否则极限一定为无穷).所 以皿(尸+7/+2)*-小贬-=寓 0+7,:T禺Q+7 3)飞+10力q所以7.讨论函数*K0+B *M0 在 x0处的连续性.解.当 a MO时fan(丁 士3*不存在,所以x=0 为第二类间断点;当时W?=0,所 以“7 时,在 X =o 连续,时,X =0 为第一类跳跃间断点.8.设 f(x)在 a,b 上连续,且 a X i x2 xn b,Ci U=1,2,3,,n)为任意正数,则在(a,b)内/(G =c j (NJ+A至 少 存 在 一 个,使 m+证明:令M=僵鸣),m=酗(%.不 妨假定/(玉

7、)Q)=U所以 m O+G+A+C*M(再)+CaZ所 以 存 在(a xi X.b),使得 CJ+CJ+A+C;9.设 f(x)在 a,b 上连续,且 f(a)b,试证在(a,b)内至少存在 一 个,使 f()二.证明:假设 F(x)=f(x)x,则 F(a)=f(a)a 0于是由介值定理在(a,b)内至少存在 一 个,使 f()二,10.设 f(x)在 0,1 上连续,且 0 f(x)1,试证在 0,1 内至少存在 一 个,使 f()二.证明:(反 证 法)反 设 V x c 0.l.f(x)=/(力-x w.所 以 V(x)=/Q)-x 恒大于0 或恒小于0.不妨设V r e 0,lL(

8、x)=/-0 令*=5 *。因 此 VxW 0.l r =/(x)-于 是/0)之1+E 1,矛盾.所以在 0,1 内至少存在一个,使 f()11.设 f(x),g(x)在 a,b 上 连 续,且 f(a)g(b),试证在(a,b)内至少存在一 个,使f()=g().证明:假设 F(x)=f(x)g(x),则 F(a)=f(a)g(a)0于是由介值定理在(a,b)内至少存在一个,使 f()=.12.证明方程十一3x2=0 在(1,2)内至少有一个实根.证明:令 F(x)=X5-3X-2,则 F(l)=-4 0所 以 在(1,2)内 至 少 有 一 个,满足F()0.1 3.设 f(x)在。呼

9、倒=。x =0的某领域内二阶可导,且*fan 八9 3,求/g/g/及 /解虱竽倒鼻x,+/W=0.所以+*)=0.f(x)在 x =0的某领域内二阶可导,所以/(*)在 x0连 续.所 以 f(o)3.因为SBfan-a-=0X,所以=0,所以/W 4-3p3向左 92*2小年用=匕.写2皿 白0上/+3_9由1 F 2,将 f(x)泰勒展开,得+热+媒力+33-1 9,所 以2 2,于是r (p)=9(本题为2 0 0 5 年教材中的习题,2 0 0 6 年教材中没有选入.笔者认为该题很好,故在题解中加入此题)倒数与微分一.填空题(理工类)i .3 =可,则,M./,g _ T-x T+,

10、_(-Di2U 产 一(-胪2*1解.0+力,。+*产,假设(1+旷1,则产(-1产2&+D I (-1/2 J d一 而严一所以J(i+x严tl+小,m一则k=.4K -/d i an A d t_2co-2m/L _ ahTcU解.M=W,k 一五居 4?丞 4?一曳=3 .设 函 数 y =y(X)由方程+830=确定,贝 I J 不一 _.解产。+/)_0,+5)3狩=0所以Ho-产/产-4 .已知f(-x)=-f(x),且 L q)=*,则/。)=.解,由 f(x)=-&)得 一/1)=-/(0所 以 0=/(幻所以6n/(勺+血)-/(-血)_5 .设 f(x)可导,则-*孩 _

11、_ _ _ _ _ _.1+血)-公)+&)-/(4-血)解.I Ax.X-+-/6).人 3-M&X:+S O -MAX=(+)/()6.设 区德节3=g则 内侮必管皿“所产以所1-3A=7.己 知 的川解-G)告二,所 以,(3=吟令一所以/GA电=8.设f为可导函数,L=(*)D,则dx .字=/3旧,(“一/5 热 力D解.&9.设y=f(x)由方程 一8 0 =-1所确定,则曲线y=f(x)在点(0,1)处的法线方程为解.上式二边求导。Q+y i-G+w)而(m=.所以切线斜率*=/(W=-2法线斜率为2,法线方程为.1-l=-x2,即 X2y+2=0.二.单项选择题(理工类)1.设

12、 f(x)可导,F(x)=f(x)(l+|sin x|),则 f(0)=0 是 F(x)在 x=。处可导的(a)充分必要条件(b)充分但非必要条件(c)必要但非充分条件(d)既非充分又非必要条件解.必 要 性:存在,所 以 久 (叽 或。于是F 8 T g PJfh=x-0 *,/(x)(t4-m JQ-/(0)Xk C/W-/(Q+/W 1X=x=r(P)+/(o)度=fS-T T o -S-xC/G O-e)-Ex=S-1=/(Q)-/(Q)所以/,(o)+/(q)=/()-/C T,2 f(0)=0,f(0)=0充分性:已知f(0)=0,所以J.-G O T(叽一人 欢14金力一0)1(

13、0)=HO*x-0 _ 3 *c/(力-o +/a)/x=;=r(P)+/(o)=r m.(Q)=*r X-0 MT X片C/W-/(O)-/(x)x所 以 0)=/(0)存在.是答案.2 .已知函数f(x)具有任意阶导数,且/Q)=/a)f,则当n为大于2的正整数时,f(x)的 n阶导数是(a)-ii/wr1(b)M/a)严(C)“co/(d)剧“户解一3=3/(*)=2/(*吐 假 设#匕)=钏/产,所以产G=(*:4-D*l/(x)f/U)=(“D D 产 按数学归纳法=l l/wr1对-切正整数成立.(a)是答案.3 .设函数对任意x 均满足f(l+x)=a f(x),且/8)=b,其

14、中a,b为非零常数,则(a)&)在*=1 处不可导(b)f(x)在 x =1 处可导,且 r0)=a(c)f(x)在 x =l 处可导,且1Q)=b (d)在*=1 处可导,且 r 0)=a b解.在 f(l+x)a f(x)中代入*=阳0=4(8=(0)=ab,所 以.(d)是答案注:因为没有假设“X)可导,不能对于/。+冷=4二边求导.4.设,8=3 +,1/1,则 使 产 8 存在的最高阶导数n为(a)0 (b)1 (c)2(d)3/W解.七:*之0 x 0/=.12*x 一 十 二励 4 8.0+。(X),所以 I Ax-.(b)是答案.6.设ax+bx0在 X=0 处可导,则(a)a

15、 =1,b =0(b)a =0,b为任意常数(c)a =0,b =0(d)a =1,b为任意常数解.在*=0 处可导一定在x =0 处连续,所以fan xsn=fan(ox+A)*x ,所以b =0.xasii 皿/,(P*)=/1(0O,S*X S r T,所以 0=a.(c)是答案.7.设 f(0)=0,则 f(x)在 x =0 处可导的充要条件为电力存在.星%在.晚 Q 供 一存在.氏 口 吁/叫 花1 0 解.由/存在可推出(a)中的极限值为9,中的极限值为(d)中的极限值为了,而(c)中的极限为:患和=&/七y=小。=。反之(a)及(c)中的极限值存在,不一定/存在,举反例如下:y

16、=I x l,不存在,(a)、(c)二表达式的极限都存在排除(a)及(c).(d)中的极限存在,不一定 r 存 在,举反例如下:10*=0,排除(d).所以(b)是答案.由(b)推 出 存 在 证 明 如 下:f g =z x=x E x 则1一 幻匕/Or f FF 1nl f-*b(t-x)*=*H l-*)所 以/存 在.8.设函数f(x)在(一,+)上可导,则(a)当把/但=7时,必 有 把 八 加 Y(b)当的/我)=时,必 有 包A)=Y(c)当 息 但=3时必 有 限 八 所 制(d)当 皋/8=种 时,必 有 区 尺)=他解.(a)不正确.反例如下:y =x;(b)不正确.反例如下:/=*;(c)不正确.反例如下:(d)是答案.证明如下:因为氏/(融=吗所以对于充分大的x,7 8单 增.如 果 融 /8=*,则证明结束,否则(单增有上界,则区存在(k 为有限数).任取x,在区间 x,x +1 上用拉格朗日定理/(x +l)-/(x)=/,(5)(x 0 且r 3)。.(d)f(a)0 i./W 0,/(0)=l0|f(x)|=f(x),在 x =0 可 导.排除(c);

展开阅读全文
相关资源
正为您匹配相似的精品文档
相关搜索

最新文档


当前位置:首页 > 商业/管理/HR > 营销创新

电脑版 |金锄头文库版权所有
经营许可证:蜀ICP备13022795号 | 川公网安备 51140202000112号